Kreisbögen

Neue Frage »

Kreisrund56 Auf diesen Beitrag antworten »
Kreisbögen
Meine Frage:
Seien zwei Punkte gegeben, durch die Kreisbögen von Kreisen mit verschiedenen Radien gehen. Dabei fällt mir auf, dass die Kreisbögen von Kreisen mit kleineren Radien stets über denen von Kreisen mit größeren Radien liegen. Woran liegt das? Kann man das sogar soweit verallgemeinern, dass wenn zwei Punkte eines Kreisbogens mit größeren Radius (r1) innerhalb eines Kreises mit kleineren Radius (r2) liegt, der gesamte Kreisbogen (r1) sich innerhalb des Kreises mit r2 befindet?

Meine Ideen:
Ich schätze mal ja
HAL 9000 Auf diesen Beitrag antworten »
RE: Kreisbögen
Zitat:
Original von Kreisrund56
Kann man das sogar soweit verallgemeinern, dass wenn zwei Punkte eines Kreisbogens mit größeren Radius (r1) innerhalb eines Kreises mit kleineren Radius (r2) liegt, der gesamte Kreisbogen (r1) sich innerhalb des Kreises mit r2 befindet?

Du meinst das in der Weise, wie es deine Skizze nahelegt: Und zwar, dass Anfangs- und Endpunkt des größeren Kreises () auf dem Rand des kleineren Kreises liegen? Dann ja.

Zum Beweis/Begründung kann man z.B. den Kreiswinkelsatz heranziehen.
Kreisrund56 Auf diesen Beitrag antworten »

Nicht unbedingt, aber wenn die zwei Punkte innerhalb des Kreises liegen, so müsste es zumindest gelten, wenn der große Kreisbogen <180° ist oder? Geht das auch mit dem Kreiswinkelsatz zu beweisen?
Bürgi Auf diesen Beitrag antworten »
RE: Kreisbögen
Guten Tag,

nur als Ergänzung

Zitat:
Dabei fällt mir auf, dass die Kreisbögen von Kreisen mit kleineren Radien stets über denen von Kreisen mit größeren Radien liegen.


[attach]55872[/attach]

Es gibt einen kleinsten Kreis der die gegebene Strecke als Durchmesser hat.
Je kleiner der Radius, desto größer ist die Krümmung, d.h., die Kreislinie "beult" sich stärker nach "außen".
Alle Kreismittelpunkte liegen auf der Mittelsenkrechten der gegebenen Strecke.
Mit wachsender Radiuslänge nähert sich die Kreislinie der gegebenen Strecke an.

..... und schon wieder weg Wink
Phenix Auf diesen Beitrag antworten »
RE: Kreisbögen
Ungleich große Kreise (R1, r2) schneiden sich, wenn …
Phenix Auf diesen Beitrag antworten »
RE: Kreisbögen
2(R1 - r2) < M1M2 < 2(R1 + r2)
 
 
HAL 9000 Auf diesen Beitrag antworten »

Zitat:
Original von Phenix
Ungleich große Kreise (R1, r2) schneiden sich, wenn …

... für den Abstand ihrer Mittelpunkte die Ungleichung gilt. Gleichheit beim linken bedeutet nur Berührung (von innen), während Gleichheit beim rechten auch Berührung (aber von außen) bedeutet.

War hier zwar nicht gefragt, aber danke fürs Interesse.


EDIT: Nur um es festzuhalten, bevor er es wieder wegeditiert:

Zitat:
Original von Phenix
2(R1 - r2) < M1M2 < 2(R1 + r2)
Phenix Auf diesen Beitrag antworten »

Völlig richtig und deshalb muß der kleinere Kreis, wenn er sich mit dem größeren schneidet, immer außerhalb des großen Kreises liegen!
… und nicht umgekehrt.
HAL 9000 Auf diesen Beitrag antworten »

Zitat:
Original von Phenix
deshalb muß der kleinere Kreis, wenn er sich mit dem größeren schneidet, immer außerhalb des großen Kreises liegen!

Du weißt schon, was du da geschrieben hast, oder? unglücklich
Kreisrund56 Auf diesen Beitrag antworten »

Um nochmal zu meiner Frage zurückzukommen. Den Fall, dass beide Punkte auf dem Kreisrand liegen konnte ich durch Hal's Tipp mit dem Kreiswinkelsatz beweisen. Allerdings bin ich mir noch unklar für den Fall, dass die Punkte innerhalb des kleinen Kreises liegen. Gilt hier wie vorhin vormutet, dass der Kreisbogen mit dem größeren Radius und <180° vollständig innerhalb das kleinen Kreises liegt?
HAL 9000 Auf diesen Beitrag antworten »

Zitat:
Original von Kreisrund56
Gilt hier wie vorhin vormutet, dass der Kreisbogen mit dem größeren Radius und <180° vollständig innerhalb das kleinen Kreises liegt?

Dann ja. Für >180° findet man jedoch leicht Gegenbeispiele.
Phenix Auf diesen Beitrag antworten »

HAL9000,
natürlich nur ein Teil des kleinen Kreises liegt außerhalb des großen Kreises, ist doch klar!
HAL 9000 Auf diesen Beitrag antworten »

Nichts ist klar, solange du so abgrundtief falsch formulierst. Und in deiner Formel oben ist Faktor 2 auch falsch.
Phenix Auf diesen Beitrag antworten »

Die ZWEI kannst du doch wegkürzen …

Es ist doch ganz einfach, bei R und r schneiden sich die beiden Kreise, oder der kleinere Kreis liegt außerhalb vom großen Kreis oder innerhalb des großen Kreises.
Kreisrund56 Auf diesen Beitrag antworten »

Hast du für diesen Fall auch eine kleine Beweisskizze? Und falls ihr noch eine 3. Meinung für die Auseinandersetzung braucht - Phenixs Beiträge haben lediglich verwirrt, anstatt irgendeine Form von Hilfestellung zu leisten
HAL 9000 Auf diesen Beitrag antworten »

So ein Unfug: Wenn man durch 2 teilt, steht in der Mitte der Doppelungleichung die halbe Entfernung der Mittelpunkte statt - wie es richtig ist - diese Entfernung selbst.

Mindestens 25% von dem, was du erzählst, ist der reinste Müll. Bei einem, der soviel postet, ist das eine erschreckend hohe Ausfallquote. Vielleicht mal mehr Qualität als Quantität, wie wäre es?

Und das schlimme ist auch noch, dass du diese offensichtlichen Fehler nicht mal einsiehst. Hat sich nichts geändert seit - ja, den erwähne ich immer wieder, weil du dich nie dafür entschuldigt hast - diesem unsäglichen Quersummenthread.
Phenix Auf diesen Beitrag antworten »

Bitte mehr Respekt und halte dich an die Fragestellung dieses Threads, das bist du dem Fragesteller schuldig!
Ich bin raus …
HAL 9000 Auf diesen Beitrag antworten »

Zitat:
Original von Phenix
Bitte mehr Respekt und halte dich an die Fragestellung dieses Threads, das bist du dem Fragesteller schuldig!

Doppelter Schwachsinn mal wieder von dir:

Zum einen hast du hier die Fragestellung des Threads verlassen mit deinem "Kreisbögen Ungleich große Kreise (R1, r2) schneiden sich, wenn … " und der dann falschen Antwort auf diese Frage. Zum anderen bist du erneut nicht bereit, fachliche Fehler zuzugeben und lenkst stattdessen wieder ab - erbärmlich. Kotzen

Zitat:
Original von Kreisrund56
Und falls ihr noch eine 3. Meinung für die Auseinandersetzung braucht - Phenixs Beiträge haben lediglich verwirrt, anstatt irgendeine Form von Hilfestellung zu leisten

So ist es.
Leopold Auf diesen Beitrag antworten »

Zitat:
Original von Phenix
Die ZWEI kannst du doch wegkürzen …


In einer falschen Formel eine 2 wegstreichen, damit die Formel stimmt, nennt man nicht kürzen, sondern korrigieren. Das ist kein innermathematische Operation, sondern eine außermathematische.
Phenix Auf diesen Beitrag antworten »

Er braucht und muss ja nicht wegkürzen, oder?
Leopold Auf diesen Beitrag antworten »

Niemand muß hier irgendetwas. HAL hätte deine falsche Ungleichung auch einfach unkommentiert stehen lassen können. Niemand wird schließlich gezwungen dazuzulernen. Jeder darf nach eigenem Gutdünken klüger werden oder dumm bleiben. HAL wollte beim Klügerwerden eine kleine Hilfestellung geben. Aber solche Hilfestellungen darf man ausschlagen. Selbstverständlich. Jeder richtet sich es so ein, wie er sich wohlfühlt. Nur darf er nicht erwarten, daß andere ihn dafür loben.
Betty65 Auf diesen Beitrag antworten »

Könnte vielleicht jemand den Beweis mitteilen, ich komme von alleine nicht darauf
HAL 9000 Auf diesen Beitrag antworten »

Ich nehme an, du meinst dies hier:

Zitat:
Original von Kreisrund56
Allerdings bin ich mir noch unklar für den Fall, dass die Punkte innerhalb des kleinen Kreises liegen. Gilt hier wie vorhin vormutet, dass der Kreisbogen mit dem größeren Radius und <180° vollständig innerhalb das kleinen Kreises liegt?

Betrachten wir den zu dem Kreisbogen gehörenden vollständigen Kreis größeren Radius. Der schneidet offenbar den kleineren Kreis, da zwei seiner Punkte im Inneren des kleineren Kreises liegen und er offenbar nicht vollständig im Inneren dieses kleineren Kreises liegen kann. Der Kreisbogen des größeren Kreises, der innerhalb des kleineren Kreises liegt, hat einen Öffnungswinkel (sonst könnte er nicht in den kleineren Kreis passen), entsprechend der Kreisbogen , der außerhalb des kleineren Kreises liegt, einen Öffnungswinkel .

Betrachten wir nun den Kreisbogen der die beiden vorgegebenen Punkte verbindet. Für den gibt es zwei Fälle:

a) Er ist Teil von , dann ist die Behauptung erfüllt.

b) Er umfasst vollständig , dann ist sein Öffnungswinkel größer als der von , und damit , was laut Voraussetzung ausgeschlossen ist.
Neue Frage »
Antworten »



Verwandte Themen

Die Beliebtesten »
Die Größten »
Die Neuesten »